Last visit was: 24 Apr 2024, 15:45 It is currently 24 Apr 2024, 15:45

Close
GMAT Club Daily Prep
Thank you for using the timer - this advanced tool can estimate your performance and suggest more practice questions. We have subscribed you to Daily Prep Questions via email.

Customized
for You

we will pick new questions that match your level based on your Timer History

Track
Your Progress

every week, we’ll send you an estimated GMAT score based on your performance

Practice
Pays

we will pick new questions that match your level based on your Timer History
Not interested in getting valuable practice questions and articles delivered to your email? No problem, unsubscribe here.
Close
Request Expert Reply
Confirm Cancel
SORT BY:
Date
Tags:
Show Tags
Hide Tags
avatar
Intern
Intern
Joined: 09 Oct 2015
Posts: 29
Own Kudos [?]: 114 [11]
Given Kudos: 15
Send PM
VP
VP
Joined: 30 Jan 2016
Posts: 1232
Own Kudos [?]: 4558 [3]
Given Kudos: 128
Send PM
Retired Moderator
Joined: 10 Oct 2016
Status:Long way to go!
Posts: 1144
Own Kudos [?]: 6119 [1]
Given Kudos: 65
Location: Viet Nam
Send PM
VP
VP
Joined: 12 Dec 2016
Posts: 1030
Own Kudos [?]: 1779 [1]
Given Kudos: 2562
Location: United States
GMAT 1: 700 Q49 V33
GPA: 3.64
Send PM
Re: In order to reduce traffic congestion and raise revenue for the city, [#permalink]
1
Bookmarks
broall wrote:
Akela wrote:
In order to reduce traffic congestion and raise revenue for the city, the mayor plans to implement a charge of $10 per day for driving in the downtown area. Payment of this charge will be enforced using a highly sophisticated system that employs digital cameras and computerized automobile registration. This system will not be ready until the end of next year. Without this system, however, mass evasion of the charge will result. Therefore, when the mayor’s plan is first implemented, payment of the charge will not be effectively enforced.

Which one of the following is an assumption on which the argument depends for its conclusion to be properly drawn?


Premise:
The mayor plans to implement a charge for driving.
Mass evasion of the charge will result unless the highly sophisticated system is used.
This system will not be ready until the end of next year.

Conclusion:
If the mayor's plan is first implemented, the payment of the charge will not be effectively enforced.

The conclusion states that the payment of charge will not be effectively enforced because of mass evasion. However, the mass evasion is due to the lack of that highly sophisticated system. This means that the mayor's plan to charge for driving will be implemented before that system is ready, or before the end of next year. Choice A states this logic well.


do you know what source of this question is?

to all test takers, using POE, A is left. Here, ones can see that an assumption is much different from a weaken. In this question, the assumption is what makes the conclusion hold true. In A, if the charge is implemented after the end of the year, then the conclusion is no longer true.
User avatar
Non-Human User
Joined: 01 Oct 2013
Posts: 17213
Own Kudos [?]: 848 [0]
Given Kudos: 0
Send PM
Re: In order to reduce traffic congestion and raise revenue for the city, [#permalink]
Hello from the GMAT Club VerbalBot!

Thanks to another GMAT Club member, I have just discovered this valuable topic, yet it had no discussion for over a year. I am now bumping it up - doing my job. I think you may find it valuable (esp those replies with Kudos).

Want to see all other topics I dig out? Follow me (click follow button on profile). You will receive a summary of all topics I bump in your profile area as well as via email.
GMAT Club Bot
Re: In order to reduce traffic congestion and raise revenue for the city, [#permalink]
Moderators:
GMAT Club Verbal Expert
6920 posts
GMAT Club Verbal Expert
238 posts
CR Forum Moderator
832 posts

Powered by phpBB © phpBB Group | Emoji artwork provided by EmojiOne